3
$\begingroup$

Has it been ruled out that the Riemann hypothesis fails, but fails only for finite number of zeros?

  • 5
    No. (Comments must be at least 15 characters.)2011-12-09
  • 5
    A related post (http://mathoverflow.net/questions/50186/if-the-riemann-hypothesis-fails-must-it-fail-infinitely-often)2011-12-09
  • 1
    @stopple, one trick is to append a dollar sign, a bunch of left and right brackets, and a dollar sign.2011-12-09
  • 7
    See?${}{}{}{}{}$2011-12-09
  • 0
    Cool!${{{{{}}}}}$2011-12-10

0 Answers 0